L'n-esima disuguaglianza, suppongo...

Polinomi, disuguaglianze, numeri complessi, ...
Rispondi
Avatar utente
HiTLeuLeR
Messaggi: 1874
Iscritto il: 01 gen 1970, 01:00
Località: Reggio di Calabria

L'n-esima disuguaglianza, suppongo...

Messaggio da HiTLeuLeR »

Va', che ogni tanto mi cimento anche di qua... :roll:

Problema #1: siano $ n $ un intero $ \geq 2 $ ed $ x_1, x_2, \ldots, x_n\in\mathbb{R}^+ $. Provare che $ \dfrac{x_1^2}{x_2^2} + \dfrac{x_2^2}{x_3^2} + \ldots + \dfrac{x_n^2}{x_1^2} \geq $ $ \dfrac{x_1}{x_2} + \dfrac{x_2}{x_3} +\ldots + \dfrac{x_n}{x_1} $, e stabilire in quali casi sussiste l'uguaglianza.
Avatar utente
NicolasBourbaki
Messaggi: 56
Iscritto il: 01 gen 1970, 01:00
Località: Pisa

Messaggio da NicolasBourbaki »

Scusatemi..sono molto di fretta quindi non scrivo le formule cmq è immediato.
1-usare QM-AM (quadrando) per modificare il primo membro
2-usare AM-GM per concludere (si osservi che i termini a secondo membro hanno sempre media geometrica 1).
Uguaglianza:iff i termini a secondo membro sono tutti uguali tra loro iff sono uguali tra loro i vari x (con indice da 1 ad n).
Speriamo che la fretta non mi sia stata troppo cattiva consigliera..
Avatar utente
Pixel
Messaggi: 79
Iscritto il: 23 feb 2005, 16:16
Località: Trento

Messaggio da Pixel »

Bah posto anche la mia anche se non sono molto sicuro, comunque sia eccovela:
Applichiamo M.Q>=M.A ed otteniamo:
$ (\frac{x_1}{x_2})^2+\dots+(\frac{x_n}{x_1})^2\geq\frac{1}{n}*(\frac{x_1}{x_2}+\dots+\frac{x_n}{x_1})^2 $ (1)
A questo punto notiamo che per la disuguaglianza di riarrangiamento si ha
$ }\frac{x_1}{x_2}+\dots+\frac{x_n}{x_1}\geq n $ ma allora
$ (\frac{x_1}{x_2}+\dots+\frac{x_n}{x_1})^{-1}\leq\frac{1}{n} $
che applicata alla (1) fornisce la tesi

Il segno uguale dovrebbe valere se e solo se $ x_1=...=x_n $ ma questo è ancora da controllare.

Ditemi che ne pensate :oops:
Ciao
P. Andrea
Avatar utente
Boll
Messaggi: 1076
Iscritto il: 01 gen 1970, 01:00
Località: Piacenza

Re: L'n-esima disuguaglianza, suppongo...

Messaggio da Boll »

Non ho ancora letto la Pixel's, ma mi pare che la disug abbia Cauchy stampato in faccia.
HiTLeuLeR ha scritto:Problema #1: siano $ n $ un intero $ \geq 2 $ ed $ x_1, x_2, \ldots, x_n\in\mathbb{R}^+ $. Provare che $ \dfrac{x_1^2}{x_2^2} + \dfrac{x_2^2}{x_3^2} + \ldots + \dfrac{x_n^2}{x_1^2} \geq $ $ \dfrac{x_1}{x_2} + \dfrac{x_2}{x_3} +\ldots + \dfrac{x_n}{x_1} $, e stabilire in quali casi sussiste l'uguaglianza.
Per Cauchy-Schwarz
$ (1+1+...+1)\left(\dfrac{x_1^2}{x_2^2} + \dfrac{x_2^2}{x_3^2} + \ldots + \dfrac{x_n^2}{x_1^2}\right)\ge \left(\dfrac{x_1}{x_2} + \dfrac{x_2}{x_3} +\ldots + \dfrac{x_n}{x_1}\right)^2 $
$ \left(\dfrac{x_1^2}{x_2^2} + \dfrac{x_2^2}{x_3^2} + \ldots + \dfrac{x_n^2}{x_1^2}\right)\ge \dfrac{1}{n}{\left(\dfrac{x_1}{x_2} + \dfrac{x_2}{x_3} +\ldots + \dfrac{x_n}{x_1}\right)^2 $
rimane da provare che
$ \dfrac{1}{n}{\left(\dfrac{x_1}{x_2} + \dfrac{x_2}{x_3} +\ldots + \dfrac{x_n}{x_1}\right)^2\ge \dfrac{x_1}{x_2} + \dfrac{x_2}{x_3} +\ldots + \dfrac{x_n}{x_1} $
$ \dfrac{x_1}{x_2} + \dfrac{x_2}{x_3} +\ldots + \dfrac{x_n}{x_1}\ge n $
che è celeberrima LINK!

Quindi l'uguaglianza se sussistono entrambe le uguaglianze, la seconda si ha sse $ x_i=x_j $$ \forall i,j $, in questo caso vale anche la prima. End
Ultima modifica di Boll il 18 giu 2005, 19:08, modificato 1 volta in totale.
Avatar utente
Pixel
Messaggi: 79
Iscritto il: 23 feb 2005, 16:16
Località: Trento

Messaggio da Pixel »

sì che è poi la mia :lol:
P. Andrea
Avatar utente
Boll
Messaggi: 1076
Iscritto il: 01 gen 1970, 01:00
Località: Piacenza

Messaggio da Boll »

Ezzi, scusa per il quasi-doppione :D
Avatar utente
Pixel
Messaggi: 79
Iscritto il: 23 feb 2005, 16:16
Località: Trento

Messaggio da Pixel »

Ma figurati :lol:

Ciao alla prossima
P. Andrea
Avatar utente
NicolasBourbaki
Messaggi: 56
Iscritto il: 01 gen 1970, 01:00
Località: Pisa

Messaggio da NicolasBourbaki »

Diciamo che è sempre la stessa zuppa rigirata in tre piatti diversi..!!
Avatar utente
HiTLeuLeR
Messaggi: 1874
Iscritto il: 01 gen 1970, 01:00
Località: Reggio di Calabria

Messaggio da HiTLeuLeR »

Ok a tutti e tre!!!
Avatar utente
HumanTorch
Messaggi: 281
Iscritto il: 01 gen 1970, 01:00
Località: Tricase

Messaggio da HumanTorch »

Se non dispiace mi inserisco anch'io, sono appena tornato e non ho letto le altre soluzioni, quindi chiedo venia per eventuali boiate:

Allora, innanzitutto dati $ n $ numeri la cui media è data, il valore minimo per $ \sum^n_{i=1} a_i^2 $ è dato quando $ a_i=a_j $ per ogni $ i,j $ distinti. La dimostrazione va da sè per induzione (ovvero, se $ n=2 $, $ (m-a)^2+(m+a)^2>2m^2 $ con $ a\neq 0 $ e così via). Quindi consideriamo il peggiore dei casi, ovvero quello appena esposto
Ma la disuguaglianza fra tutti i terminisi ottiene solo quando $ x_i=x_j $, con $ i,j $ naturali distinti (il rapporto deve essere sempre $ 1 $ altrimenti non c'è coincidenza con le altre frazioni); ma in tale caso si ha l'uguaglianza ($ n=n $); quindi per tutti i restanti casi si ha la tesi dimostrata
Avatar utente
HiTLeuLeR
Messaggi: 1874
Iscritto il: 01 gen 1970, 01:00
Località: Reggio di Calabria

Messaggio da HiTLeuLeR »

HumanTorch ha scritto:[...] dati $ n $ numeri la cui media è data, il valore minimo per $ \sum^n_{i=1} a_i^2 $ è dato quando $ a_i=a_j $ per ogni $ i,j $ distinti.
Essì, su questo punto mi ritrovo, sì... Del resto, basta giusto invocare la disuguaglianza QM-AM, tenendo conto del fatto che - per ipotesi - la media aritmetica di $ a_1, a_2, \ldots, a_n $ è fissata.
HumanTorch ha scritto:Quindi consideriamo il peggiore dei casi, ovvero quello appena esposto. Ma la disuguaglianza fra tutti i termini si ottiene solo quando $ x_i=x_j $, con $ i,j $ naturali distinti (il rapporto deve essere sempre $ 1 $ altrimenti non c'è coincidenza con le altre frazioni); ma in tale caso si ha l'uguaglianza ($ n=n $); quindi per tutti i restanti casi si ha la tesi dimostrata.
HumanTorch, in questo modo dimostri soltanto che le espressioni $ E_1(x_1, \ldots, x_n) := \dfrac{x_1^2}{x_2^2} + \ldots + \dfrac{x_n^2}{x_1^2} $ ed $ E_2(x_1, \ldots, x_n) := \dfrac{x_1^2}{x_2^2} + \ldots + \dfrac{x_n^2}{x_1^2} \leq n $ raggiungono entrambe il proprio minimo se $ x_1 = x_2 = \ldots = x_n $, supposte le variabli reali e positive. Ma d'altro canto questo non implica affatto che, in generale, sia $ E_1(x_1, \ldots, x_n) \geq E_2(x_1, \ldots, x_n) $, se $ x_1, x_2, \ldots, x_n\in\mathbb{R}^+ $. Una tal conclusione sarebbe valsa, casomai, se il minimo dell'una espressione fosse coinciso con il massimo dell'altra! Ma siccome non è questo il caso... :|
Avatar utente
enomis_costa88
Messaggi: 537
Iscritto il: 01 gen 1970, 01:00
Località: Brescia

Messaggio da enomis_costa88 »

Premetto che questa è una delle prime disequazioni su cui mi cimento..quindi nel caso probabilissimo in cui abbia sbagliato qualcosa vi prego di non aggredirmi ma di correggermi..grazie.
Pulendo i denominatori risulta:
$ \sum x_1^4*x_3^2*\dots*x_n^2 \ge \sum x_1^3*x_2*x_3^2*\dots*x_n^2 $
che dovrebbe essere vera per il bunching.
Ps si può applicare cosi(nel senso che è una sommatoria ciclica non simmetrica)???
Pps se qualcuno avesse qualche semplice disequazione da postare in modo tale da poter essere iniziato alla loro risoluzione..ne sarei molto grato.
Buona giornata Simone
ma_go
Site Admin
Messaggi: 1906
Iscritto il: 01 gen 1970, 01:00

Re: L'n-esima disuguaglianza, suppongo...

Messaggio da ma_go »

Boll ha scritto:Per Cauchy-Schwarz
$ (1+1+...+1)\left(\dfrac{x_1^2}{x_2^2} + \dfrac{x_2^2}{x_3^2} + \ldots + \dfrac{x_n^2}{x_1^2}\right)\ge \left(\dfrac{x_1}{x_2} + \dfrac{x_2}{x_3} +\ldots + \dfrac{x_n}{x_1}\right)^2 $
ehm... boll, stai cominciando a degenerare, come cinesi e vietnamiti... quella che hai scritto è cauchy-schwartz, effettivamente, però potevi molto più semplicemente chiamarla QM-AM! e che cavolo!
altrimenti, tanto vale dire che $ (x+y)^2 \le 2(x^2 + y^2) $ per jensen!
Avatar utente
Boll
Messaggi: 1076
Iscritto il: 01 gen 1970, 01:00
Località: Piacenza

Messaggio da Boll »

enomis_costa88 ha scritto:Premetto che questa è una delle prime disequazioni su cui mi cimento..quindi nel caso probabilissimo in cui abbia sbagliato qualcosa vi prego di non aggredirmi ma di correggermi..grazie.
Pulendo i denominatori risulta:
$ \sum x_1^4*x_3^2*\dots*x_n^2 \ge \sum x_1^3*x_2*x_3^2*\dots*x_n^2 $
che dovrebbe essere vera per il bunching.
Ps si può applicare cosi(nel senso che è una sommatoria ciclica non simmetrica)???
Pps se qualcuno avesse qualche semplice disequazione da postare in modo tale da poter essere iniziato alla loro risoluzione..ne sarei molto grato.
Buona giornata Simone
No, il bunching non vale ciclicamente, o almeno, se vale, non si chiama bunching, il bunching vale per le sole somme simmetriche
Avatar utente
Boll
Messaggi: 1076
Iscritto il: 01 gen 1970, 01:00
Località: Piacenza

Re: L'n-esima disuguaglianza, suppongo...

Messaggio da Boll »

ma_go ha scritto:
Boll ha scritto:Per Cauchy-Schwarz
$ (1+1+...+1)\left(\dfrac{x_1^2}{x_2^2} + \dfrac{x_2^2}{x_3^2} + \ldots + \dfrac{x_n^2}{x_1^2}\right)\ge \left(\dfrac{x_1}{x_2} + \dfrac{x_2}{x_3} +\ldots + \dfrac{x_n}{x_1}\right)^2 $
ehm... boll, stai cominciando a degenerare, come cinesi e vietnamiti... quella che hai scritto è cauchy-schwartz, effettivamente, però potevi molto più semplicemente chiamarla QM-AM! e che cavolo!
altrimenti, tanto vale dire che $ (x+y)^2 \le 2(x^2 + y^2) $ per jensen!
ghghgh, vabbè, me ne sono accorto dopo che era QM-AM, quando ci sono quadrati in giro, per me è sempre Cauchy

P.S. In effetti il modo più naturale per dimostrare Power Media è Jensen 8) :wink: :P
Rispondi